Behavioral Health EOR

Pataasin ang iyong marka sa homework at exams ngayon gamit ang Quizwiz!

What is the most appropriate intervention in suspected child abuse?

Assure the safety of the child, with hospitalization if necessary. The primary goal should be the safety and well-being of the child. Hospitalization may be the only way the clinician has to remove the child from the care of a possible abusive home if no other recourse is available due to a lack of social services and investigators.

A 26-year-old woman presents to the outpatient family practice office complaining of mood swings. She reports that she often feels intensely sad for a few hours followed by significant anxiety or irritability. During the interview, she frequently remarks that she feels she has lost herself and thinks that all people will abandon her at some point in her life. She feels detached from herself and sometimes is just "going through the motions" of her day. She reports that her friendships are usually very intense, and she typically spends almost every day with a new friend. She states she likes to get to know everything about them to try to avoid being left out when they make a new friend. At times, she will impulsively engage in risky behavior like gambling or using substances to try to keep friends. However, she reports that most friendships end in an angry outburst when she senses they are about to abandon the friendship. Which of the following personality disorders is this patient most likely suffering from? A) Antisocial personality disorder B) Borderline personality disorder C) Histrionic personality disorder D) Narcissistic personality disorder

B) Borderline personality disorder

What antipsychotic medication is associated with significant agranulocytosis but not with extrapyramidal side effects?

Clozapine (Clozaril) Clozapine is referred as an "atypical" antipsychotic agent. It can cause agranulocytosis and should be monitored with weekly CBC. This is not a first line treatment drug.

A 20 year-old male presents to the ED with complaints of palpitations and agitation, which developed suddenly while attending a party. On examination, the patient is moderately agitated and tremulous. Vital signs include a pulse of 110/minute and regular; respiratory rate 22/minute and blood pressure 160/92 mmHg. Skin is diaphoretic and pupils are dilated. What is the most likely diagnosis?

Cocaine use Cocaine, as well as amphetamines, leads to a clinical picture of increased sympathetic stimulation and dilated pupils.

What medical complication of eating disorders is due to purging (vomiting/laxative abuse)?

Electrolyte abnormalities Electrolyte abnormalities, particularly hypokalemia, hypochloremic alkalosis, and hypomagnesemia may occur as a result of purging

Who is the most likely adult to sexually abuse a child?

Family member Adults within the immediate or extended family perpetrate the most child sexual abuse, usually this is a trusted member of the family.

A 32 year-old female presents to the office with the complaint of worry which she can not control for the last six months. She tells you that she has symptoms at least four times per week consisting of sleep disturbances, difficulty concentrating and irritability. What is the most likely diagnosis?

Generalized anxiety disorder A patient needs to have symptoms for more days than not for six months or more, need 3 of 6 symptoms to diagnose generalized anxiety disorder.

A patient tells you that he is receiving special messages from the TV every night at 7:00 pm. This is an example of

Ideas of reference Ideas of reference are fixed beliefs that people are referring to you and about you through media.

What is first-line therapy for schizophrenia?

Initial pharmacologic therapy of schizophrenia should begin with one of the newer, "atypical" antipsychotic drugs, such as olanzapine, risperidone, quetiapine, ziprasidone, and clozapine because their side effect profile is significantly better than the older drugs, and they may be more effective for negative psychotic symptoms.

A patient with advanced AIDS complicated by toxoplasmosis presents with altered mental status, recent onset of seizures, and focal neurologic deficits. What diagnostic study is most helpful?

MRI of brain An MRI showing multiple isodense or hypodense ring-enhancing mass lesions is the most useful test for such a patient.

What is the leading cause of injury-related death in children between the ages of 1 and 15?

Motor vehicle injuries Motor vehicle injuries are the leading cause of death in children.

Higher doses of SSRIs are usually required in

Obsessive Compulsive Disorder Higher doses of SSRIs are needed in the treatment of OCD for a beneficial effect.

What is effective in the long-term treatment of panic attacks?

SSRI's are the initial drugs of choice for the long-term treatment of panic disorders.

You are treating a patient for her first episode of major depression. She has no medical problems and there is no family history of psychiatric disorders. In addition to psychotherapy, which class of drugs should be your first choice for the treatment of this patient?

Selective serotonin reuptake inhibitors (SSRIs) SSRIs are the first line treatment for depression because of ease of use, safety, and broad spectrum of treatment.

What laboratory test should be followed routinely every six to twelve months in patients taking lithium?

Thyroid stimulating hormone Lithium induces hypothyroidism because of the decrease in concentration of circulating thyroid hormones.

UDS Amphetamines

Time: 1-2 days (2-4 days) Detects: amphetamine molecule False +: Phenylephrine, Pseudophedrine, Ephedrein, Synephrine, Bupropion, Selegiline, Propanolol (poor specificity)

UDS LSD

Time: 1-3 days Detects: 2-oxo-3hydroxy-LSD

What laboratory test must be monitored frequently in patients who are taking clozapine (Clozaril)?

White blood cell count Leukopenia, granulocytopenia, and agranulocytosis occur in approximately 1% of patients on this medication, clozapine should not be dispensed without proof of monitoring.

The most important initial component of evaluating a patient with depressive illness is

assessment of suicidal risk. Two-thirds of all depressed patients contemplate suicide, and 10 to 15% commit suicide.

What laboratory abnormality is most commonly noted in bulimia nervosa?

hypokalemia Episodes of binge eating are followed by purging in the bulimic patient. Vomiting and laxative abuse are the most common methods of purging, leading to hypokalemia.

A divorced female patient presents for an employment physical. She states she has had a "run of bad luck" with jobs and has not been able to hold any job for longer than 2-3 months. She also states she has been arrested several times for getting into fights when she is out with the girls. She states she drinks an occasional beer, but denies any significant problems with alcohol. What laboratory findings would support your suspected diagnosis?

increased mean corpuscular volume, triglycerides, serum uric acid, and LFT's This patient most likely has alcohol abuse as evidence by her social, occupational and legal issues. Laboratory tests will reveal the presence of an elevated mean corpuscular volume, triglycerides, serum uric acid and liver function tests.

A 36 year-old woman admits that her husband has abused her for over ten years. You should inform the woman that she is at most risk for injury or death

just after leaving an abusive spouse. Women are more likely to be assaulted or murdered when attempting to report the abuse or leave the abusive relationship; up to 75% of domestic assaults occur after separation.

A child has been under treatment for attention-deficit hyperactivity disorder (ADHD). No response has occurred with behavioral adaptations. Which category of medication should this patient be given?

stimulants Stimulants, such as Ritalin, Dexedrine, and Cylert, are effective in 50 to 80% of children with ADHD.

UDS Ketamine

Time: 1-3 days Detects: Ketamine, Norketamine

UDS Marijuana

Time: 1-3 days (>1 month) Detects: 11-nor-9-carboxy-delta9-THC False +: hemp-containing food (rare); secondhand smoke does not give a false +

UDS Benzodiazepines

Time: 1-5 days, 2-30 Diazepam Detects: Oxazepam (most) False +: Oxaprozin, Sertraline

UDS Cocaine

Time: 2 days (7 days) Detects: Benzolyecgonine False +: coca tea/leaves (overall, highly specific)

A 20 year-old female presents with episodes of binge eating, overuse of laxatives, and periods of starvation. What is the best treatment option for this patient?

A SSRI is the drug of choice for the treatment of bulimia nervosa

UDS Phencyclidine (PCP)

Time: 7-14 days Detects: PCP False +: Dextromethorphan, Diphenhydramine, Doxylamine, Ketamine, Tramadol, Venlafaxine

An 18-year-old man with a history of moderate marijuana use presents to the primary care office for a urine drug screen. What is the duration of detectability for cannabis in this patient's urine? A) 1 month B) 1 to 7 days C) 2 to 3 days D) 7 to 14 days

A) 1 month

A 45-year-old man, currently employed as a janitor, presents to the clinic complaining of uncontrollable outbursts of anger. He reports having physical altercations with his wife and adult son on several occasions. He loudly complains that his last two psychiatric referrals were "no help." He raises his voice and demands efficacious treatment, stating that he is entitled to be seen before anyone else due to his position of prominence in the community. Which of the following is the next best step in treating this patient? A) Address appropriate communication and have the patient sign a contract regarding accepted communication in the office B) Begin pharmacologic treatment with a mood stabilizer such as topiramate C) Begin pharmacologic treatment with an antidepressant such as fluoxetine D) Refer the patient to a psychiatrist who will use transference-focused psychotherapy

A) Address appropriate communication and have the patient sign a contract regarding accepted communication in the office

A 35-year-old woman presents to the psychiatric clinic complaining of increased stress. She denies marital difficulties, emotional difficulties and compulsive behaviors, but states that people around her seem to have no regard for the "rules". Co-workers do not follow procedures, her children do not perform their chores in proper order, and other drivers on the road do not strictly obey traffic laws as she does. This "lack of order" causes her great stress. Which of the following is the next best step in treatment of this patient's condition? A) Administer the Working Alliance Inventory B) Begin treatment with exposure therapy C) Prescribe buspirone 10 mg twice daily D) Prescribe fluoxetine 20 mg once daily

A) Administer the Working Alliance Inventory -12 or 36 questionnaire -self-reported -used to determine goals for therapy and level of therapist-patient bond

Which segment of the brain is responsible for processing fear? A) Amygdala B) Hypothalamus C) Pineal gland D) Thalamus

A) Amygdala

A 19-year-old young woman presents to your office with a concern about her weight. She tells you that she often eats excessive amounts of food, followed by either self-induced vomiting or excessive exercise. Physical exam findings include bilateral parotid gland enlargement, periodontal disease, and abrasions on the knuckles of her hand. Which of the following laboratory test results is most likely to be increased? A) Amylase B) Calcium C) Magnesium D) Potassium

A) Amylase

UDS GHB

Time: <24 hrs Detects: GHB False +: endogenous neurotransmitter

A 42-year-old woman presents to your office with concerns about anxiety. She tells you that over the past three years she has come to realize that the local weatherman on the news channel that she watches is in love with her. She watches the news daily and reports seeing hidden references to his love throughout the weather report. She says that she ran into this man on the street and he brushed against her as he walked by, which she saw as another sign of love. She spends all day at home either watching the news, checking the news channel website or reading social media posts about this man. She is very anxious about what is going to happen regarding this relationship. Which of the following is the most appropriate therapy? A) Aripiprazole B) Clozapine C) Haloperidol D) Sertraline

A) Aripiprazole

A previously healthy 55-year-old man presents to your office requesting laboratory testing to confirm that his gastrointestinal tract has no abnormalities. The patient is a successful attorney, happily married, and convinced that the government has been trying to poison him for the past year. He refuses to eat any food in public, will not eat at a restaurant, and will not consume food prepared by others or brought by coworkers. He is concerned that the government may now be starting to poison food at the supermarket. He wonders if it will be safe to continue eating food at his house. Which of the following is the most appropriate therapy? A) Aripiprazole B) Citalopram C) Clozapine D) Fluoxetine

A) Aripiprazole

A 4year-old boy presents in clinic with his parents for evaluation of poor performance in preschool. His parents state he has never maintained eye contact, did not start walking until he was two years old, and has a restricted vocabulary but appears to understand most commands. Additionally, they state he has very specific interests and only plays with toy trains and toy clocks. He has one older brother, age 7 years, who has a normal developmental history and no significant medical history. The patient does not interact with his brother despite his brother's attempts to engage in play. On exam, he is well-nourished and well-developed, with no gross neurological abnormality. He avoids eye contact and does not answer questions. He prefers to sit alone in a chair with his toy clock. What is the most likely diagnosis? A) Autism spectrum disorder B) Conduct disorder C) Oppositional defiant disorder D) Social communication disorder

A) Autism spectrum disorder

Which of the following symptoms, if present in a patient more days than not for six months or more, would aid in the diagnosis of generalized anxiety disorder? A) Being easily fatigued B) Decreased appetite C) Fear of having a panic attack D) Preoccupation with previous traumatic events

A) Being easily fatigued

A 24-year-old man is brought into the ED by a friend who states the man is "acting crazy" and is fighting with an imaginary person. The patient's behavior became erratic after he ingested what appeared to be a pill. He appears disoriented and agitated. He took a swing at the security guard on his way into the building but lost his balance. Physical exam shows both vertical and horizontal nystagmus. What medication is the first-line therapy to treat this patient's intoxication? A) Benzodiazepines B) Neuroleptics C) Opioid antagonist D) Sodium bicarbonate

A) Benzodiazepines

A 30-year-old man with no chronic conditions presents to the clinic as a new patient concerned about his mood and energy level. He has felt persistently sad, hopeless, and tired for the past two years. He cannot think of any life event that triggered these symptoms. Physical examination is unremarkable. Which of the following diagnostic studies is indicated? A) Blood analysis B) Brain magnetic resonance imaging C) Polysomnography D) Thyroid ultrasonography

A) Blood analysis

A 23-year-old man presents to your clinic with concerns about low back pain and his medication not working any more. He says that two years ago he had an injury at work that resulted in ongoing back pain. Right after the accident, one of his medical providers gave him a prescription for oxycodone, which helped at first, but then he noticed having to take more than the prescribed amount in order to have the same relief of his pain symptoms. His medical provider declined to continue prescribing oxycodone, so he started to buy it off the street. Recently a friend of his offered him heroin, which he tried once. This scared him because he liked it so much that he wanted to continue using it, but he knows that this can lead to serious problems. Which of the following is the most appropriate therapy? A) Buprenorphine/naloxone B) Duloxetine C) Extended release oxycodone D) Gabapentin

A) Buprenorphine/naloxone

A 25-year-old woman with a history of bulimia nervosa presents to your office with concerns about depression. She has started psychotherapy and would like to start taking a medication as well. Which of the following is contraindicated for use in this patient? A) Bupropion B) Fluoxetine C) Nortriptyline D) Sertraline

A) Bupropion

Which of the following medications used in the management of anxiety has a delayed onset of action? A) Buspirone (BuSpar) B) Diphenhydramine (Benadryl) C) Lorazepam (Ativan) D) Butalbital (Fiorinal)

A) Buspirone (BuSpar) Buspirone takes several days to weeks for it to have clinical activity.

A 12-year-old boy is starting a methylphenidate patch to treat his attention-deficit/hyperactivity disorder. Which of the following should be performed prior to initiating treatment? A) Cardiac evaluation B) Hormone level evaluation C) Liver function testing D) Thyroid evaluation

A) Cardiac evaluation

Which of the following factors is associated with an increased risk of recurrence of major depressive disorder after treatment? A) Childhood maltreatment B) No prior exposure to adversity C) Older age at time of initial symptoms D) Substance misuse

A) Childhood maltreatment

A 44-year-old man, previously diagnosed with panic disorder, presents to the clinic stating that he is unwilling to continue taking medication for his panic disorder. He states he cannot tolerate the side effects, does not want to take any more pills, and is very open to other methods of treatment. Which of the following would be the best choice for initial, non-pharmacologic treatment of his panic disorder? A) Cognitive-behavioral therapy B) Electroconvulsive therapy C) Hypnotherapy D) Repetitive transcranial magnetic stimulation

A) Cognitive-behavioral therapy

A 20-year-old man presents to the emergency department with a bilateral loss of sensation in his hands. The patient states that he witnessed a motor vehicle collision that involved several fatalities. He reports he observed the incident but was not physically involved in the collision. The patient has no previous history of neurological problems or mental illness. On examination, the patient appears in mild emotional distress but is indifferent to the loss of sensation in his hands. He is concerned about missing work and asks to be discharged as soon as possible. The remainder of the physical exam is normal, including a full neurological workup. A psychological evaluation reveals that the patient is not intentionally feigning illness. Which of the following is the most likely diagnosis? A) Conversion disorder B) Factitious disorder C) Malingering D) Somatic symptom disorder

A) Conversion disorder

A 38-year-old woman presents to the clinic complaining of "chronic scabies". She states she has been to many clinics, received treatment for scabies multiple times, and yet continues to be infested. She states the parasites are even beginning to crawl around in her eyeballs. There are excoriations present on her upper extremities. Besides being distressed by her illness, she does not complain of depression, hallucinations, or cognitive impairment. She is steadily employed, denies any substance use, and has no psychiatric history. Which of the following is the most likely diagnosis? A) Delusional disorder B) Schizophrenia C) Schizotypal personality disorder D) Widespread scabies

A) Delusional disorder

A 17-year-old previously healthy young woman presents to your office with her mother who has parental concerns about her weight. Her mother says that the patient has been steadily losing weight, is very focused on her appearance, and spends a lengthy amount of time in the bathroom several times a week. Which additional physical exam finding would suggest a diagnosis of bulimia nervosa rather than anorexia nervosa? A) Dental caries B) Hypotension C) Low body mass index D) Thinning hair

A) Dental caries

A 26-year-old woman presents in the emergency department reporting severe anxiety, tremors, confusion, nausea, sweating, and palpitations. She reports misusing lorazepam and alprazolam for two years. She reports last taking lorazepam three days ago. Diaphoresis and significant bilateral hand tremors are noted on exam. Her heart rate is 130 beats per minute. Which of the following is the most appropriate therapy? A) Diazepam B) Flumazenil C) Propranolol D) Valproate

A) Diazepam -long-acting Benzodiazepine indicated for withdrawal *Flumazenil is given for acute Benzodiazepine intoxication

A 35-year-old woman presents to her primary care office with a three-month history of dull headaches, insomnia, and irritability. She has difficulty concentrating and says she is disinterested in her favorite activities. She states she has been having nightmares and frequent flashbacks of her time overseas. The patient recently left the military and is currently employed as a nurse at a local hospital. What important historical feature has she provided that distinguishes her symptoms as post-traumatic stress disorder rather than acute stress disorder? A) Duration B) Headaches C) Insomnia D) Nightmares

A) Duration

A 19-year-old man presents to clinic for intermittent mood swings, coughing, headaches, nausea, and loss of appetite. His mother states he often emerges from the family's home office or his bedroom with these symptoms, which usually resolve within a few hours. Additionally, she states he has an unusual odor when the symptoms occur. Which of the following physical exam finding is likely due to the substance being abused? A) Erythematous rash about the mouth B) Erythematous streaks on the antecubital fossa C) Severe tooth decay with gingivitis D) Skin abscesses

A) Erythematous rash about the mouth

A 40-year-old man presents to the clinic to discuss his engagement in and preoccupation with unusual sexual activities that have been occurring for the past few years. The patient reports he was recently arrested for indecent exposure. He states that he exposed his genitals to an adult woman at the beach and was excited by her shocked reaction. He says he has done this before and states it is usually followed by him masturbating. Which of the following is the most likely diagnosis? A) Exhibitionistic disorder B) Frotteuristic disorder C) Sexual masochism disorder D) Voyeuristic disorder

A) Exhibitionistic disorder

A 20-year-old woman presents to the clinic complaining of an inability to keep food down which has become increasingly worse over the past six months. She also complains of cold intolerance, amenorrhea, constipation, and thinning of her hair. She states she watches what she eats very closely and spends a significant amount of time calculating her daily caloric intake. Her body mass index is currently 15 kg/m2. Which of the following physical examination findings would be expected? A) Fine, downy, unpigmented hair growth B) Hyperactive bowel sounds C) Hyperthermia D) Oily skin

A) Fine, downy, unpigmented hair growth

A 35-year-old woman presents to your office with a concern about fear of flying. She says that she has never liked to fly, but over the past three years this fear has increased in intensity. She says that thinking about flying or even seeing an airplane results in feelings of severe anxiety and panic. She has not flown in over five years because of this fear. Her family is having a reunion next month on the other side of the country and she would like to attend, but is afraid that she won't be able to fly because of her anxiety. Which of the following is the most appropriate therapy? A) Lorazepam B) Paroxetine C) Sertraline D) Venlafaxine

A) Lorazepam

Which of the following best describes the mechanism of action of delta-9-tetrahydrocannabinol (THC), the psychoactive substance in marijuana? A) Partial agonist at the cannabinoid 1 and cannabinoid 2 receptors B) Partial agonist at the cannabinoid 2 receptor C) Pure agonist with high affinity at cannabinoid 1 receptor and moderate affinity at cannabinoid 2 receptor D) Pure agonist with very high affinity at the cannabinoid 1 receptor

A) Partial agonist at the cannabinoid 1 and cannabinoid 2 receptors

A patient with obsessive-compulsive disorder would most likely have which of the following findings? A) Raw, red hands B) Priapism C) Memory impairment D) Abdominal pain

A) Raw, red hands Common manifestations of obsessive-compulsive disorder include phobias of germ and contaminants, which results in frequent hand washing leading to chafe and reddened hands. The other answers are inconsistent with obsessive-compulsive disorder.

A 9-year-old boy presents to the office with his mother because he is often losing his temper at home. He is argumentative and refuses to listen when she asks him to do his chores. Additionally, he is making fun of his brother to the point where his brother no longer wants to be around him. He is always blaming his brother for making him mad. Upon further questioning, you find out that the behavior began worsening about 6 months ago and seems worse than the usual behavior for his age. She reports that her children spend a lot of time at home alone and she is often nagging or yelling at them to behave. What type of clinical intervention would work best for this family to address the patient's behavior? A) Refer the family to parent management therapy that includes the patient and his mother B) Refer the patient to group therapy with other boys and girls his age C) Start the patient on a low-dose selective serotonin reuptake inhibitor for six weeks D) Start the patient on a stimulant such as methylphenidate

A) Refer the family to parent management therapy that includes the patient and his mother

A 20-year-old woman presents to the clinic for a sports physical. Her body mass index is 15 kg/m2. She reports restricting her diet for the past 3 months to 800 calories per day and running at least 10 miles each day. She states she wants to be healthy and fears being out of shape. Which of the following would lead to a diagnosis of anorexia nervosa in this patient? A) The patient believes she is currently overweight B) The patient has lanugo and acne C) The patient is competing for a spot on the cross country team D) The patient states she bloats and has intense abdominal pain after each meal

A) The patient believes she is currently overweight

A 45 year-old female presents with her daughter who is concerned because her mom will not leave the house. The patient describes feeling anxious in crowds and places where there are no easy means of escape. What is the most likely diagnosis in this patient?

Agoraphobia Though all of the anxiety disorders can have comorbid agoraphobia the case describes only those symptoms inherent to this primary diagnosis. Agoraphobia can occur with generalized anxiety disorder and a panic attack can occur as a result of agoraphobia.

Formications are most commonly associated with

Alcohol withdrawal Formications are the sensation of insects crawling on the skin and is commonly associated with delirium tremens from alcohol withdrawal and cocaine addiction.

What are the most effective agents in treating somatoform spectrum pain disorder?

Antidepressants Antidepressant medications often help with both the pain and the worry surrounding the pain.

Early clues to impending delirium tremens include

Anxiety, decreased cognition, tremulousness, increasing irritability, and hyperactivity are common early clues to impending delirium tremens.

A mother brings her 6 year-old boy for evaluation of school behavior problems. She says the teacher told her that the boy does not pay attention in class, that he gets up and runs around the room when the rest of the children are listening to a story, and that he seems to be easily distracted by events outside or in the hall. He refuses to remain in his seat during class, and occasionally sits under his desk or crawls around under a table. The teacher told the mother this behavior is interfering with the child's ability to function in the classroom and to learn. The mother states that she has noticed some of these behaviors at home, including his inability to watch his favorite cartoon program all the way through. What is the most likely diagnosis?

Attention deficit hyperactivity disorder Attention deficit hyperactivity disorder is characterized by inattention, including increased distractibility and difficulty sustaining attention; poor impulse control and decreased self-inhibitory capacity; and motor overactivity and motor restlessness, which are pervasive and interfere with the individual's ability to function under normal circumstances.

A mother brings in her five year-old boy for his school physical. She voices some concerns about his readiness for school, saying he seems to be socially immature. She has noticed he does not interact with other children well, and that when he plays with them, he has a tendency to "place them" and then run around them as if they were statues. He rarely cries when he is hurt, and he shrugs off any attempt to hug him. He has good attention to details, and will sit and draw the same geometric shapes over and over again, but does not seem interested in learning the alphabet. He avoids eye contact with anyone. What is the most likely diagnosis?

Autism Children with autism do not tend to make eye contact, and even avoid it. They do not accept comfort when hurt and stiffen up when hugged. They do not tend to play with others, and do not tend to imitate grown-ups in play. They approach play in a more mechanical way, using others as props rather than interacting with them.

A 50-year-old woman returns to her primary care physician for the fourth time in the past 30 days. Today, she states she has a cough and thinks it may be pneumonia or cancer. She has looked up the symptoms of both online. On physical exam, she has no noticeable cough, her lungs are clear to auscultation bilaterally, and no fever is present. Last week she came in with complaints of bleeding easily. The week prior, she had been taking an excessive amount of aspirin because she was sure she was going to have a stroke. She has tried cognitive-behavioral therapy and psychotherapy without success. What class of medication would be most effective to manage her illness? A) Antibiotic B) Antidepressant C) Antipsychotic D) Antitussive

B) Antidepressant

A 7-year-old boy presents to the office with his mother for evaluation of his angry and defiant behavior that has been steadily worsening over the past year. The mother is teary and exasperated, explaining that it is a battle every day to get him to do normal tasks like getting ready for school or eating meals since he is often argumentative and does not do what he is told. He constantly blames her and her husband for making his life miserable. You suspect oppositional defiant disorder. Assessing for the presence of what common co-occurring disorder is key in effective treatment? A) Antisocial personality disorder B) Attention-deficit/hyperactivity disorder C) Autism spectrum disorder D) Disruptive mood dysregulation disorder

B) Attention-deficit/hyperactivity disorder About half of children with ADHD have ODD.

A 27-year-old woman presents to your office requesting a referral for psychotherapy. She tells you that she is constantly fighting with her current partner, has not been able to maintain a romantic relationship for more than three months, has not been able to maintain employment because "they treat me unfairly", has a history of self mutilation by cutting and has multiple suicide attempts in the past. Which of the following is the most likely diagnosis? A) Anxiety disorder B) Borderline personality disorder C) Histrionic personality disorder D) Paranoid personality disorder

B) Borderline personality disorder

A 20-year-old previously healthy woman with a body mass index of 20 kg/m2 presents to your office for her annual exam. She tells you that she is very concerned about her weight and has been weighing herself daily. Vital signs reveal a blood pressure of 92/65 mm Hg and pulse of 102 bpm. Physical exam findings include bilateral parotid gland enlargement, dental caries, and abrasions on the knuckles of her right hand. Which of the following is the most likely diagnosis? A) Anorexia nervosa B) Bulimia nervosa C) Major depressive disorder D) Obsessive-compulsive disorder

B) Bulimia nervosa

A 15-year-old boy is brought to your office by his mother. She states over the past year he has been repeatedly in trouble at school for engaging in or starting fights. He is frequently truant and has subsequently been held back from advancing to 10th grade. He has also been arrested for stealing at a local mall. The patient denies that any of his mother's concerns are valid and says "everyone else is doing the same stuff". Which of the following do you suspect? A) Attention deficit hyperactivity disorder B) Conduct disorder C) Intermittent explosive disorder D) Oppositional defiant disorder

B) Conduct disorder

A 16-year-old boy has a two-year history of defiant behavior toward authority figures. His parents report that he is constantly getting into fights at school. They feel he has no respect for the rules in their house and will purposefully break rules to upset them. He has caused thousands of dollars in house repairs due to angry outbursts that lead him to punch holes in walls or kick down doors. He has had legal issues related to stealing merchandise from stores. He denies feeling irritable or depressed. A screening for psychotic symptoms is negative. What is the most likely diagnosis? A) Attention-deficit hyperactivity disorder B) Conduct disorder C) Intermittent explosive disorder D) Oppositional defiant disorder

B) Conduct disorder -anger could be consistent with intermittent explosive disorder -other symptoms more specific to conduct

A 23-year-old woman is complaining of insomnia. She states that she is unable to sleep during the day or night because she is certain the local police are watching her house and monitoring her location at all times. She began noticing different cars parked outside of her house about two months ago and realized they must be watching her. She is very organized in her speech pattern. She is expressive in describing how she tries to cleverly avoid being seen by them when leaving her home to visit her sister who lives nearby. She denies hallucinations and has no history of previous psychiatric diagnoses. What is the most likely diagnosis? A) Brief psychotic disorder B) Delusional disorder C) Schizoaffective disorder D) Schizophrenia

B) Delusional disorder

Which of the following is an example of a negative symptom associated with schizophrenia? A) Decreased reactivity to the environment B) Diminished speech output C) Displaying childlike silliness D) Loosening of associations

B) Diminished speech output (all others are results of positive symptoms) Decreased reactivity to the environment- catatonic behavior Displaying childlike silliness- disorganized behavior Loosening of associations- disorganized thinking and speech

A 25-year-old man is brought to the ED by his father because of a change in behavior. For the past three weeks, the patient has been locking himself in the bathroom. The patient states that the IRS is after him for his secret to the universe. His speech is disorganized and rambling. He appears very anxious and is convinced that this is part of a plot to poison him. He is becoming fearful and extremely agitated. His father states that this has never occurred previously. A decision to administer intramuscular haloperidol is made as the patient is refusing to take oral medication. What medication should be given with the haloperidol to reduce the risk of severe extrapyramidal symptoms including dystonias? A) Chlorpromazine B) Diphenhydramine C) Lorazepam D) Olanzapine

B) Diphenhydramine Diphenhydramine or Benztropine minimize the risk of severe extrapyramidal symptoms

A 44-year-old man presents to the clinic with involuntary facial movements. He states that for the past several months, his lower lip and jaw move in writhing motions every half hour or so, and it is beyond his control. He is taking risperidone 2 mg daily. When questioned about the reason for taking risperidone, he states that his previous healthcare provider put him on the drug because he has no friends (and doesn't want any), has little facial expression, finds no pleasure in any activity, and prefers to be alone. He denies any past history of visual or auditory hallucinations. Which of the following therapeutic changes would be best to make for this patient? A) Decrease the dose of risperidone and add clonazepam B) Discontinue risperidone and refer for psychotherapy C) Discontinue risperidone and replace with clozapine D) Discontinue risperidone and replace with fluoxetine

B) Discontinue risperidone and refer for psychotherapy -misdiagnosed with schizophrenia -correct diagnosis: schizoid personality disorder -Schizoid PD will not have positive symptoms of schizophrenia TOC for personality disorders: psychotherapy

A 35-year-old woman was recently diagnosed with tobacco use disorder and is being treated with bupropion. The drug acts by inhibiting which of the following processes? A) Binding of nicotine to α-4 β-2 nicotinic receptor B) Dopamine and norepinephrine reuptake C) Norepinephrine and serotonin reuptake D) Presynaptic serotonin reuptake

B) Dopamine and norepinephrine reuptake

A 24-year-old woman presents to the clinic complaining that last month she suffered two episodes of intense fear of dying with palpitations, chest pain, chills, dizziness and shortness of breath. The episodes came on rapidly and resolved after twenty minutes. Which of the following statements would help diagnose panic disorder in this patient? A) Fearful of another episode, the patient has avoided crowds, strangers and parties for the past month B) Fearful of another episode, the patient has avoided travelling more than five miles from home for the past month C) Fearful of becoming contaminated and having another episode, the patient has avoided the outdoors for the past month D) Fearful of losing her husband, the patient has accompanied him to work for the past month

B) Fearful of another episode, the patient has avoided travelling more than five miles from home for the past month

A 45-year-old man with a family history of early-onset coronary artery disease avoids climbing stairs and partaking in exercise. He abstains from sexual activity for fear of provoking a heart attack. He frequently checks his pulse, reads extensively about preventive cardiology, and has tried many health food supplements alleged to be good for the heart. When he experiences an occasional twinge of chest discomfort, he rests in bed for 24 hours; however, he does not go to the doctor because he fears hearing bad news about his heart. What diagnosis best fits this clinical picture? A) Generalized anxiety disorder B) Illness anxiety disorder C) Panic disorder D) Somatic symptom disorder

B) Illness anxiety disorder

Which of the following best describes the post-traumatic stress disorder self-report checklist known as the PCL-5? A) A score of ten is associated with a diagnosis of post-traumatic stress disorder B) It can be used for screening purposes and to monitor symptom severity over time in post-traumatic stress disorder C) The checklist contains 100 questions, each with a total value of zero to four points D) Two different versions are currently used, which distinguish between civilian and military populations

B) It can be used for screening purposes and to monitor symptom severity over time in post-traumatic stress disorder PCL-5: 22 question self-report questionnaire effective in identifying PTSD and monitoring changes in symptom severity over time

A 55-year-old woman with a history of schizoaffective disorder presents to your office to discuss her current treatment regimen. She was started on Olanzapine at her last visit and wants to know what type of monitoring is recommended while taking this medication. Which of the following is the most appropriate diagnostic test to order? A) Absolute neutrophil count B) Lipid profile C) Renal panel D) Thyroid stimulating hormone

B) Lipid profile 2nd gen antipsychotics increase risk of metabolic syndrome due to side effects of weight gain, dyslipidemia, and DM.

Which of the following best represents an indication for use of electroconvulsive therapy as first-line treatment in bipolar I disorder? A) Episode of mania with insomnia B) Major depressive episode with malignant catatonia C) Major depressive episode with suicidal ideation without a plan D) Manic episode with severe flight of ideas

B) Major depressive episode with malignant catatonia (due to: life-threatening nature of the illness) Malignant cataonia: immobility or waxy flexibility with fever, rigidity, and signs of autonomic instability

Which of the following is one of the eight criteria needed for the diagnosis of post-traumatic stress disorder? A) Duration of the disturbance associated with the traumatic event is more than two months B) Marked alterations in mood and cognition associated with and beginning after the traumatic event C) Persistent pursuance of stimuli related to the traumatic event, beginning after the event has occurred D) Presence of two or more intrusion symptoms associated with the traumatic event and beginning after the event has occurred

B) Marked alterations in mood and cognition associated with and beginning after the traumatic event 1. Exposure to actual or threatened death, serious injury, or sexual violence 2. 1 or more intrusive symptoms associated with the event 3. Persistent avoidance of stimuli 4. Negative alterations in cognition and mood related to the event 5. Marked alterations in arousal and reactivity 6. Duration >1 month 7. Disturbance causes clinically significant distress or impairment in important functioning 8. Disturbance cannot be attributed to substance use or another medical condition

A 26-year-old man presents to your office for evaluation. His substance abuse screening tool is positive. Vital signs include a blood pressure reading of 90/71 mm Hg and respiratory rate of 10 breaths per minute. Physical exam reveals altered mental status and track marks on his arms. Which of the following additional findings would suggest a diagnosis of opioid intoxication rather than stimulant intoxication? A) Elevated temperature B) Miosis C) Mydriasis D) Tremor

B) Miosis

A 33-year-old woman presents to the clinic complaining of excessive worry which is difficult to control and has persisted for the past year, accompanied by difficulty sleeping and irritable mood. Which of the following additional signs or symptoms most closely suggests generalized anxiety disorder? A) Cognitive impairment B) Muscle tension C) Tearfulness D) Weight loss

B) Muscle tension

A 25-year-old man presents to the clinic complaining of anxiety. He states that he greatly fears that he will get lost while driving. For this reason, he repeatedly checks his GPS device, even while driving routes with which he is very familiar. He has been in four motor vehicle collisions this past year as a result of checking his GPS device instead of watching the road. Which of the following is the most likely diagnosis? A) Generalized anxiety disorder B) Obsessive-compulsive disorder C) Paranoid personality disorder D) Specific phobia

B) Obsessive-compulsive disorder

A 50-year-old man presents to the office with a complaint of a white patch to the top of the tongue that has slowly increased in size over the past six months. The patch is painless, does not bleed, and does not impede chewing or eating. The patient has a 30-pack-year history and consumes alcohol on social occasions. Upon examination, you note a 1 x 2 cm white patch on the superior portion of the tongue that has a rough texture and well-defined irregular borders. It does not come off with scraping and does not bleed when manipulated. What is the most likely diagnosis? A) Oral candidiasis B) Oral leukoplakia C) Oral lichen planus D) Squamous cell carcinoma of the tongue

B) Oral leukoplakia

What condition is defined as the occurrence of repeated panic attacks that are associated with intense, debilitating fear of future panic attacks? A) Generalized anxiety disorder B) Panic disorder C) Posttraumatic stress disorder D) Specific phobia

B) Panic disorder

A 23-year-old woman presents to the clinic complaining of feeling sad and hopeless for the past three years. She reports that she wants to sleep almost all of the time and that she has been overeating. She has never sought medical attention for these symptoms before. Which of the following options is the most likely diagnosis? A) Major depressive disorder B) Persistent depressive disorder C) Premenstrual dysphoric disorder D) Seasonal affective disorder

B) Persistent depressive disorder

After a thorough evaluation and physical exam, a 38-year-old woman is diagnosed with cyclothymic disorder. Which of the following clinical characteristics must be present to make this diagnosis? A) Symptoms are concurrent with substance abuse B) Symptoms are never gone for more than two consecutive months C) Symptoms are present continually for two years D) Symptoms do not cause serious psychosocial impairment

B) Symptoms are never gone for more than two consecutive months

A 20-year-old woman presents to the clinic complaining of excessive fear of embarrassment and humiliation, which has caused her to isolate herself from her peers. This has worsened over the past three years. In situations where she is unable to avoid interaction with others, she notes that she sweats and feels as though her hands are shaking. She has been unable to obtain a part-time job and struggles to participate in group activities in school because of this. Which additional finding would suggest a diagnosis of social anxiety disorder rather than agoraphobia? A) The fear causes clinically significant distress B) The fear is due to the presence of people C) The fear is not better explained by another medical condition D) The fear is of being in public places where escape may be difficult

B) The fear is due to the presence of people

What is the most appropriate intervention for a patient suffering from a specific phobia, such as fear of snakes?

Behavioral therapy Specific phobias most commonly are treated with behavioral therapy including exposure therapy utilizing systemic desensitization. Hypnosis, supportive therapy and family therapy may also be useful adjunct treatment.

What class of medications has been found most beneficial in the treatment of alcoholism not associated with a concomitant psychiatric illness?

Benzodiazepines Though no pharmacologic therapies have substantial supportive data in treating alcoholism not associated with a mood or anxiety disorder short term use of benzodiazepines has the greatest benefit especially with acute cessation of alcohol.

A patient presents with an episode of an expansive, elevated mood during which she cleaned excessively without sleeping. What is the most likely diagnosis?

Bipolar disorder Bipolar disorder is characterized by episodic mood shifts from depression to manic type moods which is often rapid with depression lasting longer than manic episodes. Bipolar disorder may initially present with a manic episode.

A 30-year-old woman presents to her primary care physician's office with a history of difficulty paying attention, forgetfulness, poor time management skills, and impulsiveness dating back to her childhood. Which of the following is the most likely diagnosis? A) Alcohol use disorder B) Anxiety disorder C) Attention deficit hyperactivity disorder D) Bipolar disorder

C) Attention deficit hyperactivity disorder

A 30-year-old otherwise healthy woman taking no medications presents to the clinic complaining of occasional episodes of increased energy, decreased need for sleep, expansive mood, easy distractibility, shopping sprees, and rapid speech. These episodes last four or five days at a time and then resolve. In the past year, she has experienced two such episodes. She reports no hallucinations, delusions, hospitalizations, impaired social functioning, or substance use but was treated for major depression five years ago. Which of the following is the most likely diagnosis? A) Attention deficit hyperactivity disorder B) Bipolar I disorder C) Bipolar II disorder D) Cyclothymic disorder

C) Bipolar II disorder S/S consistent with hypomania because they did not lead to hospitalization or cause major impairments on social functioning and does not contain elements of psychosis.

A 28-year-old woman presents with symptoms of depression. She states that she has had mood swings during the past 2 years. Upon eliciting a history, she describes multiple mild episodes of hypomanic symptoms and mild depression that have caused her distress. She does not recall a period of time over the last several years when these symptoms were not present. She reports no substance use. Which of the following is the most likely diagnosis? A) Bipolar I disorder B) Bipolar II disorder C) Cyclothymic disorder D) Schizoaffective disorder

C) Cyclothymic disorder Multiple episodes of hypomanic symptoms (w/o diagnosis) and episodes of mild depression (w/o major depressive diagnosis) over at least 2 years with no less than 2 months without symptoms.

A 20-year-old woman presents to the clinic complaining of stress. During the interview, she taps her fingers repetitively on her leg in a specific pattern. When questioned, she states she taps them because she has a "bad thought" about hurting someone, and each time she thinks it, she has to tap her fingers in a specific manner to erase the thought. Which of the following questions, if answered in the affirmative, would be most likely to aid in the diagnosis of obsessive-compulsive disorder in this patient? A) Do you feel the bad thought is put there by someone else? B) Does the compulsive behavior cause you anxiety? C) Does the compulsive behavior cause you social or occupational problems? D) Does the thought of harming others bring you enjoyment?

C) Does the compulsive behavior cause you social or occupational problems?

A 27-year-old woman presents to the clinic for a refill of her fluoxetine, which she is taking for generalized anxiety disorder. Most of her worry centers around the welfare and safety of her children, and she states the fluoxetine is helping to control her anxiety to some extent. In addition to encouraging her to continue taking her medications correctly and seeing her mental health therapist on a regular basis, which of the following best represents a behavioral strategy that should be encouraged in this patient? A) Avoid anxiety-provoking situations B) Avoid exposure to catastrophic images C) Engage in progressive muscle relaxation D) Engage in repeated checking on children while they sleep

C) Engage in progressive muscle relaxation

A 45-year-old previously healthy woman presents to your office with a complaint of depressed mood. She says that over the past six months she has had difficulty sleeping, poor appetite, fatigue, lack of interest in her usual social activities, and recurrent episodes of crying. On physical exam she has lost 15 pounds since her last office visit and her hygiene is poor. She has never taken medication or had treatment for depression. Which of the following is the most appropriate therapy? A) Amitriptyline B) Aripiprazole C) Escitalopram D) St. John's Wort

C) Escitalopram

A 25-year-old woman presents to the emergency department with complaints of abdominal pain, vomiting, and diarrhea that has been present for five years. She states that no one has been able to help her up to this point. She has numerous allergies to a variety of medications. Physical exam reveals moist mucous membranes, multiple well-healed incisions on her abdomen, and vital signs within normal limits. Which of the following is the most likely diagnosis? A) Borderline personality disorder B) Conversion disorder C) Factitious disorder D) MalingeringYour Answer

C) Factitious disorder Key aspects: symptoms inconsistent with physical exam findings, evidence of multiple episodes of care, and inconsistent stories Why not malingering? Malingering would include some sort of personal gain

Which of the following medications is indicated as a first-line treatment for mild to moderate body dysmorphic disorder? A) Bupropion B) Citalopram C) Fluoxetine D) Mirtazapine

C) Fluoxetine

An 8-year-old boy is brought to your office by his parents for evaluation because they are concerned with his behavior at school. His teacher reports that he is constantly getting out of his seat during lessons, cannot sit still, and is constantly disrupting the class by speaking out of turn. What features of this child's history would point to the diagnosis of attention-deficit/hyperactivity disorder? A) He acts out at school but not at home for at least one year B) He has been acting out at school for at least three months C) He has been acting out both at school and at home for at least six months D) His sister also has similar behavior at school

C) He has been acting out both at school and at home for at least six months

A 28-year-old woman presents to the primary care office for a routine check-up. While observing her in the waiting room, you note that she is telling a very loud and dramatic story which has captivated the other patients. She becomes annoyed when she has to stop telling her story to be taken into the exam room. She is noted to be dressed in tight pants, a mesh shirt with a hot pink bra, and three inch high heels. During her interview, she relates that she has gone through another three boyfriends since her last visit three months ago. She begins to cry inconsolably because she was convinced that each one was going to be her future husband and was genuinely surprised that they each broke up with her. She can provide few details about her relationships. However, she suddenly becomes overjoyed when she starts to talk about an upcoming singles cruise she recently booked at the suggestion of a checkout clerk at the grocery store. Which of the following personality disorders does this patient most likely have? A) Borderline personality disorder B) Dependent personality disorder C) Histrionic personality disorder D) Narcissistic personality disorder

C) Histrionic personality disorder

A 17-year-old girl is diagnosed with anorexia nervosa. She has fallen below 70 percent of her ideal body weight and is hospitalized for the initial stages of nutritional replenishment. What is the hallmark and predominant cause of refeeding syndrome? A) Congestive heart failure B) Hypokalemia C) Hypophosphatemia D) Thiamine deficiency

C) Hypophosphatemia During episodes of starvation, phosphate stores are depleted. When carbohydrates are reintroduced during aggressive nutritional supplementation, insulin triggers cellular uptake of phosphate which results in low serum phosphate levels. Extreme hypophosphatemia can lead to myocardial dysfunction and respiratory failure. Volume overload can also occur from increased renal sodium causing fluid retention. Monitoring for and correcting hypophosphatemia, hypokalemia, and hypomagnesemia is important during nutritional rehabilitation.

A 26-year-old woman presents to a psychiatrist for cognitive behavioral therapy for fear of driving. She has not yet received a driver's license because she is in constant fear of getting into an accident once behind the wheel. Upon further questioning, she states she had a cousin who was killed in a motor vehicle collision. Which of the following factors best describes how this patient likely developed her fear of driving? A) Cognitive bias B) Direct conditioning C) Informational transmission D) Vicarious acquisition

C) Informational transmission

A 45-year-old man presents to the clinic with a lack of desire for sexual activity, which has been present for the last seven months and is subsequently diagnosed with male hypoactive sexual desire disorder. This disorder is a result of the absence of which of the following male sexual functions? A) Ejaculation B) Erection C) Libido D) Orgasm

C) Libido

A 34-year-old man presents to the outpatient primary care office with his wife for evaluation. The wife explains that she is concerned about her husband's recent preoccupation with building a vintage Corvette. She came home two weeks ago to find him in the garage with car parts scattered around the yard and driveway. He spent approximately $23,000 on these parts, but justifies this behavior by reminding her how much profit he will make when completes and sells the Corvette. She is doubtful of this outcome, considering her husband knows very little about cars and mechanics. She decided to bring him today after he failed to pay the mortgage this month or show up to work for the last week. He seems unconcerned about either of these things. She reports that he has been overly happy since taking up this new hobby, but it is difficult to get him to focus on any one particular topic in conversation as he seems to be going "a million miles a minute". He has no other past medical history and takes no other medications. Which of the following medications should be started for this patient? A) Bupropion B) Lamotrigine C) Lithium D) Mirtazapine

C) Lithium (TOC for acute mania) -others: anticonvulsants and antipsychotics *Lamotrigine for bipolar 1 maintenance*

Which of the following is a necessary component for the diagnosis of major depressive disorder? A) Depressed mood in response to an identifiable stressor, within three months of onset of the stressor B) Depressed mood most of the day, nearly every day for at least two consecutive months C) Loss of interest or pleasure in most activities, nearly every day for at least two consecutive weeks D) Thoughts of worthlessness or excessive guilt nearly every day for at least one month

C) Loss of interest or pleasure in most activities, nearly every day for at least two consecutive weeks

Which of the following medications is first-line therapy for males with pedophilic disorder? A) Bupropion PO B) Fluoxetine PO C) Medroxyprogesterone IM D) Methylprednisolone IM

C) Medroxyprogesterone IM

A 12-year-old boy presents to clinic with a history of conduct disorder. His parents report he is having increased high-risk behavior despite behavioral and psychological therapy. They are concerned about his safety. Which of the following medications is the best choice for treating patients with a diagnosed conduct disorder? A) Citalopram B) Lithium C) Methylphenidate D) Valproic acid

C) Methylphenidate (stimulants are TOC for conduct disorder)

A 35-year-old woman enters a crowded restaurant. She approaches the hostess and demands immediate seating although there are people waiting ahead of her. She states that she is very important in local politics and needs to be seated immediately. The hostess respectfully explains that there are a few patrons ahead of her and that she will be put on the list. The woman becomes enraged and hostile. She asks to see the manager and complains that she is so important that she should be seated first regardless of those ahead of her and believes they would understand because of who she is. She states that the hostess is probably just jealous and that is why she won't seat her. According to the DSM-5, what is the most likely diagnosis? A) Borderline personality disorder B) Histrionic personality disorder C) Narcissistic personality disorder D) Paranoid personality disorder

C) Narcissistic personality disorder

Which of the following childhood exposures increases the risk of conduct disorder? A) Alcohol in utero B) Aspirin use C) Neglect D) Tetracycline use

C) Neglect Conduct risk factors: -neglect and abuse -fam hx of psychiatric illness -parental substance abuse -parental-marital conflict

A 26-year-old woman presents to the outpatient family practice office with a past medical history significant for schizoaffective disorder. She reports that she is dead already. She also believes that a 7.0 magnitude earthquake is going to occur in seven days. She describes that she was reading a gardening magazine three weeks ago when she was struck with the knowledge that she no longer exists and that a natural disaster is around the corner. She has been unable to focus on anything else. She denies any visual or auditory hallucinations. When presented with the information that she was present in the office and there has never been a recorded earthquake where she lives, she remains undeterred from this thought. How should this information be documented in the history of present illness? A) Erotomanic delusions B) Grandiose delusions C) Nihilistic delusions D) Persecutory delusions

C) Nihilistic delusions

A 22-year-old woman presents to the clinic complaining of loss of appetite, increased anxiety and dental caries. She displays increased energy and irritability, increased talkativeness, excoriations of the extremities, tachycardia, pupillary dilatation and diaphoresis. Which of the following is the most likely next step in diagnosis of this patient? A) Administer the Mood Disorder Questionnaire B) Obtain a thyroid-stimulating hormone level C) Obtain a urine toxicology screen D) Question the patient about past episodes of mania or depression

C) Obtain a urine toxicology screen

A 26-year-old woman presents to the clinic with a four month history of depressed mood, feelings of worthlessness, hypersomnolence, and 20 pound weight gain. She has also noticed changes in the texture of her hair and nails, an inability to concentrate, and a leaden feeling in her limbs. She has no suicidal or homicidal ideation. Which of the following would be the most appropriate next step in diagnosis of this patient's condition? A) Administer a Mini-Mental State Examination B) Have the patient complete a Patient Health Questionnaire-9 C) Order a complete metabolic panel and serum thyroid stimulating hormone level D) Refer the patient to a psychiatrist for evaluation

C) Order a complete metabolic panel and serum thyroid stimulating hormone level

A 22-year-old man presents to the emergency department by emergency transport. His girlfriend reports he uses amyl nitrite recreationally. On exam, there is significant cyanosis, a clear airway, and good respiratory effort. He remains cyanotic despite oxygen delivery at 10 liters per minute through a non-rebreathing mask. Lab analysis confirms methemoglobinemia. Which of the following reactions mediates the formation of methemoglobinemia from amyl nitrite exposure? A) Dephosphorylation B) Glycolysis C) Oxidation D) Phosphorylation

C) Oxidation

A 24-year-old man presents with dysphoria, loss of interest in daily activities, insomnia, and feelings of worthlessness. These symptoms have been present for the last three years and he has been going to a support group without improvement in symptoms. Which of the following is the best initial treatment for this patient? A) Electroconvulsive therapy B) Pharmacotherapy alone C) Pharmacotherapy and psychotherapy D) Psychotherapy alone

C) Pharmacotherapy and psychotherapy

A 24-year-old woman presents to clinic for follow-up on anorexia nervosa. She has gained four pounds in the last six months and currently has a body mass index of 19 kg/m². In addition to routine medical examination and dietary assessment, what other treatment component must be included in relapse prevention? A) Antihistamines B) Antipsychotics C) Psychotherapy D) Selective serotonin reuptake inhibitors

C) Psychotherapy

A 23-year-old woman is hospitalized with her first episode of acute mania. She is treated and stabilized with quetiapine 300 mg daily. She is sexually active and uses condoms irregularly. Upon discharge, which of the following represents the best choice of maintenance therapy for this woman? A) Psychotherapy alone B) Psychotherapy and lithium 300 mg TID C) Psychotherapy and quetiapine 300 mg daily D) Psychotherapy and valproate 250 mg TID

C) Psychotherapy and quetiapine 300 mg daily

A 45-year-old homeless man is brought to the emergency department by ambulance because he had a sudden tonic-clonic seizure in the middle of the street. A witness who walks by the patient daily noted that he was acting strangely prior to the seizure. She said that he is usually very friendly and greets her as she walks by, but this morning he seemed very agitated, had trembling hands and was yelling as if having a shouting match with himself. Upon physical examination, the patient is awake but muttering inaudibly and is not oriented to self, time, or place. His blood pressure is 198/102 mm Hg, and heart rate is 125 bpm. He appears to be malnourished and dehydrated. What detail in his history would confirm your diagnosis? A) Prior history of untreated schizophrenia B) Recent cessation of seizure medication C) Recent history of alcohol cessation D) Recent overconsumption of alcohol

C) Recent history of alcohol cessation

A 25-year-old man presents to the clinic for follow-up after being diagnosed with alcohol use disorder of moderate severity. He was recently discharged from the hospital after being involved in a motor vehicle collision. During his stay, he expressed a desire to stop drinking and completed a medical detox. His last alcoholic drink was 10 days ago. Prior to discharge from the hospital, he was started on naltrexone 50 mg per day by mouth, which he is tolerating well and continues to take daily without issue. Which of the following is the most appropriate next step in management? A) Change naltrexone to acamprosate B) Change oral naltrexone to depot naltrexone C) Recommend psychosocial treatment D) Recommend residential treatment

C) Recommend psychosocial treatment

Which of the following antipsychotics is approved for use in an adolescent diagnosed with autism? A) Haloperidol B) Pimozide C) Risperidone D) Thioridazine

C) Risperidone

A 40-year-old man arrives via EMS to the emergency department. He was found wandering on the side of the road "talking to God". He is talking rapidly and switching topics quickly. He is also complaining about all of the "spiders crawling around the room". He recently quit his seventh job in two years because he was tired of being criticized. His records show that he was brought in a few weeks ago with similar behavior. He states he has been "off his meds" for a while now because he has been feeling fantastic. Which of the following would you diagnose? A) Bipolar II disorder B) Paranoid personality disorder C) Schizoaffective disorder D) Substance abuse disorder

C) Schizoaffective disorder

A 32-year-old woman presents for a complete physical examination. During the exam, she says she has diminished interest in daily activities. Upon further questioning, she also reports feelings of worthlessness, hypersomnia, fatigue, and the inability to concentrate at work. These symptoms have been ongoing for the last month. She has never had symptoms like this in the past. Which of the following would be the most appropriate pharmacologic treatment at this time? A) Amitriptyline B) Bupropion C) Sertraline D) Tranylcypromine

C) Sertraline

A 45-year-old man presents to the clinic for his annual physical. He says he is an "alcoholic." Further questioning reveals a history of excessive alcohol use despite losing his wife and his job, cravings for alcohol, withdrawal symptoms when alcohol cessation was attempted, using alcohol to stave off withdrawal symptoms, repeated attempts to quit drinking, and drinking large amounts of alcohol for longer periods of time than intended. The patient states that he joined a support group this year and has not had an alcoholic beverage in six months. Which of the following is the appropriate diagnosis? A) Alcohol abuse in early remission B) Alcohol abuse in sustained remission C) Severe alcohol use disorder in early remission D) Severe alcohol use disorder in sustained remission

C) Severe alcohol use disorder in early remission Mild: 1-3 features Moderate: 3-5 features Severe: >5 features Early remission: within 3-12 months Sustained remission: >12 months

A 22-year-old man with a history of anxiety presents to your office with a complaint of three days of increased sweating, anxiety, nausea, and mild hand tremors. He is a college student and has a considerable amount of stress related to exams and extracurricular activities. He has been taking alprazolam 0.5 mg at bedtime for the past six months but stopped filling his prescription when he ran out about a week ago because he did not want to become dependent. He states that he has been trying other methods of helping his anxiety including exercise, yoga, and has been seeing a therapist at his school. Upon physical examination, the patient has slightly dilated pupils and a pulse of 115 bpm, but he is alert and oriented to self, time, and place with normal speech and reasoning. What is the best next step for treatment of this patient's symptoms? A) Admit the patient for close monitoring and intravenous benzodiazepine therapy to relieve his symptoms B) Refill patient's prescription of alprazolam and tell him that this is lifelong therapy and he should never try to discontinue it again C) Switch patient to a benzodiazepine with a longer half-life and taper him off slowly over the next few weeks D) Tell patient that his symptoms will abate within the next few days and that no further intervention is necessary

C) Switch patient to a benzodiazepine with a longer half-life and taper him off slowly over the next few weeks

An 8-year-old boy presents to the clinic demonstrating an angry demeanor. His mother complains that he has been angry almost every day for the past seven months. She states he often argues with her and with his teachers and disregards direct orders. Which of the following additional behaviors is most consistent with this disorder? A) Depressed mood B) Distractibility C) Vindictiveness D) Violence

C) Vindictiveness

A 35-year-old woman presents to the clinic complaining of increased irritability and decreased concentration for the past ten days. She also notes that she feels rested after four hours of sleep for the past ten nights, and has received two speeding tickets in the past week. On exam, the patient is unable to sit still and exhibits pressured speech and easy distractibility. Which of the following additional elements of her history would help lead to a diagnosis of bipolar I disorder? A) She has been married for 12 years and has been employed with the same company for 15 years B) She has not been hospitalized for any reason C) he is experiencing increased energy and has completed 350 pages of her "memoirs" in the past ten days D) Two months ago she felt "down" and stayed in bed for seven days

C) he is experiencing increased energy and has completed 350 pages of her "memoirs" in the past ten days

A 36 year-old patient presents requesting something to help him sleep. He reports that he has always had a problem sleeping, admits to feeling nervous most days for the last 2 years, and that he has always been "uptight" and a "worry wart." During the previous eight months he has frequently felt tense, shaky, sweaty, with palpitations and frequent headaches. He reports being irritable with his 5 year-old son. What is the best treatment option for this patient?

Certain antidepressants, including SSRIs (eg, escitalopram, starting dose of 10 mg po once/day) and serotonin-norepinephrine reuptake inhibitors (eg, venlafaxine extended-release, starting dose 37.5 mg po once/day) are effective but typically only after being taken for at least a few weeks. Benzodiazepines in small to moderate doses are also often and more rapidly effective, although sustained use may lead to physical dependence. One strategy involves starting with concomitant use of a benzodiazepine and an antidepressant. Once the antidepressant becomes effective, the benzodiazepine is tapered. Buspirone is also effective; the starting dose is 5 mg po bid or tid. However, buspirone can take at least 2 wk before it begins to help.

A 52 year-old female complains of bouts of anxiety and depression, the latter very deep but short lived (<24 hours). She states "I often feel like I am going to jump out of my skin". Her periods have become less frequent over the past 6 months. She denies suicidal ideations. She is not sleeping through the night. What is the most appropriate next step in this patient?

Check a follicle-stimulating hormone level An FSH level should be assessed in this patient to evaluate for menopause as a cause for her psychiatric symptoms.

What behavior would be most typical for a patient diagnosed with schizoid personality disorder?

Chooses solitary activities Persons with schizoid personalities are very withdrawn and do not seek or enjoy relationships and are indifferent to praise or criticism. They generally appear cold and unfeeling to others.

A 38 year-old female is brought to the emergency room with a complaint of paralysis of her arms bilaterally. Her history is significant for a sexual assault two days previously. Her physical examination is unremarkable with normal reflexes. There is no apparent physical explanation for her paralysis. What is the most likely diagnosis?

Conversion disorder Patients have abrupt loss of motor or sensory function caused by psychological factors that often precede the physical symptoms

Which of the following best describes a barrier to treatment for a patient with paranoid personality disorder? A) Aggression B) Agoraphobia C) Delusions of persecution D) Fragile self-concept

D) Fragile self-concept

Which of the following laboratory abnormalities would most likely be seen in a patient with severe anorexia nervosa? A) Increased creatinine B) Increased follicle-stimulating hormone C) Increased glomerular filtration rate D) Increased total cholesterol

D) Increased total cholesterol (due to increased production of cardioprotective HDL)

In order to diagnose a major depressive episode, five or more symptoms have to be present within a two-week period and have to represent a change from the previous baseline. Which of the following must be included within those five symptoms to secure a diagnosis? A) Diminished ability to concentrate B) Feelings of worthlessness C) Insomnia or hypersomnia D) Loss of interest in daily activities

D) Loss of interest in daily activities (or depressed mood)

A 13-year-old boy and his parents present to the clinic to discuss persistent behavior problems over the past year. He began with breaking items in the home and has since started fights at school and set his sister's toys on fire. He recently made threats to hurt the family dog. He is not responding to the parents' disciplinary efforts. In addition to psychotherapy, which of the following medications would be most beneficial for this patient? A) Alprazolam B) Citalopram C) Haloperidol D) Methylphenidate

D) Methylphenidate (stimulants TOC for conduct)

A 35-year-old man presents to his primary care physician's office for a physical exam. He offers no complaints other than feeling like all of his coworkers are out to get him. He denies any auditory or visual hallucinations. His physical exam is unremarkable. Which of the following should be ordered on this patient as part of his work-up? A) Computed tomography of the head B) Electroencephalogram C) Lumbar puncture D) Thyroid-stimulating hormone

D) Thyroid-stimulating hormone -no diagnostic for delusional disorder -r/o physiological causes

A 35-year-old man presents to the clinic for his annual physical. He was a project manager for a construction company until a few weeks ago. He was fired for failing to complete multiple projects on time, which he states is because every detail needs to be perfect prior to submission. He disliked delegating tasks to his co-workers as he feels they were unable to complete things his way. At home he has been having some marital difficulties due to his preoccupation with saving as much as he can. Which of the following is the most likely diagnosis? A) Borderline personality disorder B) Narcissistic personality disorder C) Obsessive-compulsive disorder D) Obsessive-compulsive personality disorder

D) Obsessive-compulsive personality disorder

A 25-year-old man presents to the emergency department complaining of restlessness, runny nose, diarrhea, vomiting, and profuse sweating. Which of the following substances is likely producing this patient's withdrawal symptoms? A) Alcohol B) Cannabis C) Lorazepam D) Oxycodone

D) Oxycodone S/S of opioid withdrawal: increased lacrimation, mydriasis, sweating, abdominal pain, diarrhea, and vomiting

A 34-year-old woman presents to the outpatient family practice office for evaluation. During her interview, she reports that she feels depressed and that this has been her baseline for the last eight or nine years. During this time, she has experienced occasional periods of happiness lasting one to two weeks at the most before returning to a sad or depressed state. She reports a generally low self esteem and indecisiveness which has made career advancement difficult. What is the most likely diagnosis? A) Bipolar I disorder B) Bipolar II disorder C) Major depressive disorder D) Persistent depressive disorder

D) Persistent depressive disorder -at least 2 years of depressed mood on most days in adulthood

A 40-year-old woman presents to your clinic with complaints of continued distress six months after she was mugged on the street at gunpoint. She says that since the event, she has recurrent thoughts and dreams of what happened, anxiety when she passes the location where it occurred, negative thoughts about others, decreased interest in activities, insomnia and flashbacks. Which of the following is the most likely diagnosis? A) Acute stress disorder B) Anxiety disorder C) Major depressive disorder D) Post-traumatic stress disorder

D) Post-traumatic stress disorder

Which of the following drugs is used to decrease nightmares and sleep disturbance in post-traumatic stress disorder? A) Amlodipine B) Clonidine C) Hydralazine D) Prazosin

D) Prazosin

Which of the following is first-line therapy for a patient with factitious disorder? A) Antidepressant medication B) Antipsychotic medication C) Electroconvulsive therapy D) Psychotherapy

D) Psychotherapy

A 24-year-old woman presents to the clinic with depressed mood, lack of appetite, decreased sleep, lack of interest in pleasurable activities, and thoughts of guilt for the past two weeks. These symptoms occur every day, for most of the hours of the day, and are causing her distress. Her family history is significant for a father with major depressive disorder and a mother with bipolar I disorder. Which of the following is the next best step in diagnosis of this patient's condition? A) Administer a Patient Health Questionnaire - 9 item B) Perform a complete metabolic panel C) Perform a urine toxicology screen D) Question the patient about previous manic episodes

D) Question the patient about previous manic episodes *always r/o bipolar disorder 1 with new MDD diagnosis*

A 42-year-old woman presents to the clinic as a new patient with numerous soft tissue injuries that she claims occurred during intercourse with her current sexual partner. She has willingly participated in sexual behavior where a partner feigned control over her for years, but lately her injuries have become so severe that they interfere with her job. She lives alone and feels capable to continue caring for herself. She denies feeling depressed or suicidal. Which of the following interventions is best for this patient? A) Admit for inpatient psychiatric care B) Refer to neurology for limbic leucotomy C) Refer to psychiatry for electroconvulsive therapy D) Refer to psychology for cognitive-behavior therapy

D) Refer to psychology for cognitive-behavior therapy (TOC of sexual masochism disorder) Medication: SSRI, Lithium, mood stabilizers, anti-androgens, Phenothiazines, long-acting GnRH *even with treatment prognosis is poor due to management often being ineffective*

A 32-year-old man with hypertension and a recent diagnosis of schizophreniform disorder presents to the clinic complaining of a persistent generalized restless feeling for the past week. He has a hard time sitting still, which is interfering with his work. His medications include losartan, amlodipine, and risperidone, which he has been increasing the dose of at the direction of his psychiatrist. Which of the following is the most likely cause for his chief complaint? A) Amlodipine B) Losartan C) Restless leg syndrome D) Risperidone

D) Risperidone

An anxious, 55-year-old woman presents to a new provider with complaints of stomach pain, fatigue, and dizziness. She cannot describe the pain well and states that it comes and goes throughout the day regardless of food intake, diet, or time of day. Three weeks ago, she was concerned about her symptoms and went to the hospital where she had an endoscopy with biopsy and other testing, but left against medical advice when the results came back negative. She reluctantly shares her records from multiple previous health care providers for the same or similar symptoms. She has spent countless hours online researching her various symptoms. No diagnosis has ever been definitively made despite the testing. The patient completes a Patient Health Questionnaire - 15 Item (PHQ-15) and scores a 20 out of 30. What would be the next best step in her treatment? A) Empirically prescribe antibiotics for Helicobacter pylori B) Repeat endoscopy with biopsy C) Schedule laparoscopic surgery D) Schedule regular outpatient visits regardless of symptoms

D) Schedule regular outpatient visits regardless of symptoms

A 21-year-old man presents to the clinic for a physical. While obtaining a detailed history, the patient reveals that last year he suffered a "mental breakdown" where he heard voices, believed his deceased grandmother was following him everywhere, had trouble communicating and would make up new words, had academic problems due to memory deficits, and felt apathetic and without energy. These symptoms lasted for six weeks and then resolved completely. The patient denies current or past substance use and also denies any other psychiatric or developmental disorders. Which of the following is the most likely diagnosis? A) Schizoaffective disorder B) Schizoid personality disorder C) Schizophrenia, first episode, currently in full remission D) Schizophreniform disorder

D) Schizophreniform disorder

A 32-year-old disheveled man, wearing a construction hat, presents to the clinic for a rash. He believes the rash was "sent" to him by a neighbor who hates him and put a curse on him. He knows the neighbor hates him because he feels a dark aura follow the neighbor around. He lives alone because he fears any roommate will rob him while he sleeps. He wears a construction hat at all times, to protect his head from falling objects. While speaking, he often searches for words and seems to daydream. He is employed from home, takes no medications, does not use drugs or alcohol, has no previous psychiatric diagnoses, denies depression, and answers questions appropriately. Family members state he has always been "like this". Which of the following is the most likely diagnosis? A) Paranoid personality disorder B) Schizoid personality disorder C) Schizophrenia D) Schizotypal personality disorder

D) Schizotypal personality disorder Characterized by: extreme discomofrt and avoidance of close personal relationships and social situations, distortions in thoughts and perceptions, and oddities of behavior

A 38-year-old man presents to the clinic complaining of excessive worry for the past six months, which is gradually worsening and causing him distress. His anxiety lasts for most of the day almost every day and is interfering with his ability to go to work, church, and the grocery store. He has no comorbid illnesses, is taking no medications and is using no other substances. Of the following choices, which would be the most appropriate therapy to initiate in this patient? A) Amitriptyline, at a starting dose equal to that which would be used in major depressive disorder B) Amitriptyline, at a starting dose half that which would be used in major depressive disorder C) Sertraline, at a starting dose equal to that which would be used in major depressive disorder D) Sertraline, at a starting dose half that which would be used in major depressive disorder

D) Sertraline, at a starting dose half that which would be used in major depressive disorder

A 28-year-old man presents to the clinic for his annual physical. The patient has a history of schizophrenia, which has been well managed with the medication olanzapine. Which of the following is the most appropriate screening test? A) Complete blood count with differential B) Electrocardiogram C) Fundoscopic exam D) Serum lipid panel

D) Serum lipid panel (and blood glucose) Olanzapine (atypical antipsychotic) is associated with significant weight gain and adverse metabolic effects (weight gain, DM, hypercholesterolemia)

Which of the following statements best describes a contributing factor in the pathogenesis of panic disorder? A) Patients who demonstrate hypoactivity in the amygdala and hypothalamus are at increased risk for development of panic disorder B) Patients who suffer narcissistic personality disorder are at increased risk for development of panic disorder C) Patients with low anxiety sensitivity are at increased risk for the development of panic disorder D) Smoking in childhood increases the risk of development of panic disorder as an adult

D) Smoking in childhood increases the risk of development of panic disorder as an adult (and childhood exposure to sexual or physical abuse)

A 55-year-old woman with a history of depression and anxiety presents with the complaint of dizziness, vague stomach pain, and generalized weakness that began when she lost her job a year ago. She states that she has been to multiple providers, including several specialists. They have failed to find anything that could be causing the symptoms. She states that she has been to the hospital on several occasions but has left prior to discharge when the tests and imaging come back negative. She has been overly concerned about her health issues and has not been looking for employment as a result. What screening test would be the most helpful in facilitating a diagnosis at this time? A) Alpha-fetoprotein blood test B) Colonoscopy C) Low-dose helical computed tomography D) Somatic Symptom Scale-8

D) Somatic Symptom Scale-8

Two years ago, a 62-year-old woman developed a deep vein thrombophlebitis and subsequent pulmonary embolism after a transatlantic airplane flight. She has a long history of chronic anxiety. Over the last two years, she has focused relentlessly on sensations of pleuritic chest pain and repeatedly seeks medical attention for this symptom. Despite negative test results, she worries that her pain is due to a recurrent pulmonary embolism. A review of her history reveals chronic low back pain and frequent dysuria which has prompted her to see multiple physicians for culture negative cystitis. What is the most likely diagnosis? A) Factitious disorder B) Illness anxiety disorder C) Malingering D) Somatic symptom disorder

D) Somatic symptom disorder *illness anxiety disorder would be the same but without the history of actually having a DVT*

A 21-year-old woman presents to the clinic for a work physical. She states she has never had blood work done and refuses to have any labs drawn today because she is fearful of needles. She reports avoiding clinics and hospitals for fear of seeing needles and is feeling very anxious, diaphoretic, and nauseous during her current visit. What is the most likely diagnosis? A) Agoraphobia B) Generalized anxiety disorder C) Panic disorder D) Specific phobia

D) Specific phobia

Which of the following is an important routine assessment for patients with factitious disorder? A) Fasting serum glucose B) Glasgow coma scale C) Magnetic resonance brain imaging D) Suicide risk

D) Suicide risk

A 65-year-old patient has a long history of schizophrenia that is treated with phenothiazines. On an unrelated clinic visit, the patient has difficulty sticking out her tongue, facial tics, increased blink frequency, and lip-smacking behavior. These involuntary movements are most suggestive of which of the following? A) Gilles de la Tourette's syndrome B) Huntington's disease C) Schizophrenia D) Tardive dyskinesia

D) Tardive dyskinesia

Chronic antipsychotic use is associated with the development of which syndrome? A) Palmar-plantar erythrodysesthesia B) Red man syndrome C) Stevens-Johnson syndrome D) Tardive dyskinesia

D) Tardive dyskinesia

A 30-year-old man who was recently arrested is brought to the jail health care clinic for intake evaluation. He says he has been arrested multiple times, with his first arrest occurring at 15 years old after vandalizing his school auditorium. He states he has always had problems with authority figures and has a hard time getting along with his peers. He has never been able to keep a job for longer than a few weeks, which is why he resorted to stealing. He does not regret any of his actions. He reports no use of any illicit drugs but binge drinks occasionally. What is the best way to diagnose this patient's condition? A) Brain magnetic resonance imaging with contrast B) Psychopathy checklist C) The Minnesota multiphasic personality inventory D) The patient's history

D) The patient's history

A 30-year-old man, currently being treated with paroxetine daily for generalized anxiety disorder, presents to the clinic complaining of episodes of tachycardia, sensations of impending doom, chest pain, and shortness of breath over the past six months. The episodes are triggered by anticipation of certain stressful events and resolve themselves within 15 minutes. Which of the following would be the most appropriate evaluation to perform when considering the diagnosis of panic disorder in this patient? A) Erythrocyte sedimentation rate, C-reactive protein, and rheumatoid factor laboratory tests B) Mini-Mental State Exam assessment C) Panic Disorder Severity Scale assessment D) Thyroid-stimulating hormone, complete metabolic panel, and complete blood count laboratory tests

D) Thyroid-stimulating hormone, complete metabolic panel, and complete blood count laboratory tests -r/o physiologic causes of panic attacks

A 26-year-old woman with posttraumatic stress disorder presents to the clinic for a medication follow-up. She was started on paroxetine two weeks ago and denies any side effects with the medication. In addition to continuing her medication, the most appropriate next step is the initiation of which of the following? A) Interpersonal therapy B) Mindfulness-based stress reduction C) Psychodynamic psychotherapy D) Trauma-focused cognitive-behavioral therapy

D) Trauma-focused cognitive-behavioral therapy

A patient presents to the outpatient psychiatric clinic after being referred for evaluation of cyclothymic disorder. What duration of time must the patient have experienced symptoms in order to meet the DSM-V criteria for this disorder? A) Four days B) One week C) Two weeks D) Two years

D) Two years

Which of the following is an example of a positive symptom of schizophrenia? A) Anxiety B) Apathy C) Impaired memory D) Word salad

D) Word salad

A 25 year-old male on a behavioral medicine unit is given haloperidol (Haldol) IM for a violent psychotic outburst. Initially he quiets down, but about an hour later develops confusion, an inability to open his mouth, and a temperature of 40 degrees C. Initial treatment should consist of

Dantrolene or Bromocriptine In addition to supportive treatment, the most commonly used medications for neuroleptic malignant syndrome are dantrolene (Dantrium) and bromocriptine (Parlodel).

A patient is evaluated for extreme fears of abandonment and an inability to care for himself throughout his adult life. He avoids disagreements and has difficulty initiating projects or acting on his own thoughts and ideas. Advice and reassurance are sought for even minor daily details. There are no reported suicidal tendencies or signs of self harm. What is the most likely personality disorder?

Dependent These patients are constantly seeking external support and will do even unpleasant things for others to gain approval and nurturing. Narcissistic patients are egotistic and would not seek the opinion of others. Patients with borderline disorder can share some of these traits as they make aggressive efforts to avoid abandonment but the suicidal tendencies, impulsivity and self mutilation differentiate it. Obsessive-compulsive patients do not rely on external support and obey very strict personal rules of perfection and efficiency and choose not to rely on others.

A 63 year-old retired engineer presents with one month of difficulty sleeping. He has a hard time staying asleep and says he is just restless. He also states that he has been more forgetful and can't pay attention very well. He lives with his wife. He denies fever, chills, recent trauma, or difficulty walking. When asked about the specifics of his symptoms, he repeatedly replies, "I don't know," without really trying. He states his wife thinks he just sits around the house all day since he retired. His neurological exam is unremarkable, except for some mild psychomotor retardation. What is the most likely diagnosis?

Depression Depression often presents with difficulty thinking and concentrating, lessened sleep, and withdrawal from activities.

What medication is the treatment of choice for benzodiazepine intoxication?

Flumazenil (Romazicon) Flumazenil is the treatment of choice for benzodiazepine intoxication.

A 24 year-old female presents to your office for a physical examination. She is dressed in a low cut blouse and a short skirt. She is dramatic, emotional and sexually provocative. She complains of difficulty being intimate with men. On further questioning, she seems to overemphasize the severity of her current cold. After a full history and physical examination you suspect what personality disorder?

Histrionic Patients who are histrionic are attention seekers, and exaggerate their thoughts and feelings, they are often sexually provocative.

A patient with schizophrenia states that newspapers and the television are constantly talking about him though they never mention his name. What term best fits this description?

Ideas of reference Ideas of reference are part of the abnormal thought content that is hallmark in schizophrenic patients. Cosmic identity refers to a patient's belief that they have physically and mentally fused with the universe while thought broadcasting is the belief that others can read the patient's mind or that their thoughts are broadcast over the radio or similar media. A distorted memory filtered through a person's present emotional state refers to retrospective falsification.

A 35 year-old male patient comes back to the office for a follow-up visit. He remarks that after 5 weeks on fluoxetine (Prozac) 20 mg per day, he still feels depressed but he denies suicidal ideations. What should you do to help this patient?

Increase the dose of fluoxetine An antidepressant should be raised to the recommended level and maintained at that level for 4-5 weeks, this patient was on too low of a dose.

A 26 year-old female arrives in the emergency department with friends who say she was standing in front of her church, dressed in a white bathrobe, claiming to be the Virgin Mary and handing out $100 bills to all passers-by. Her friends noted that she had been depressed lately, but now seems completely euphoric. She had a similar episode two years ago. What is the most appropriate treatment?

Inpatient olanzapine (Zyprexa), Lithium, or Valproate therapy Treatment of the manic phase is usually done in the hospital to protect patients from behaviors associated with grandiosity (spending inordinate amounts of money, making embarrassing speeches, etc.). Lithium, valproate, and olanzapine are considered effective in the manic stage; the depressive stage is treated with antidepressants.

A 25 year-old female presents to the emergency department after she collapsed at work. Medical history is significant for a mood disorder that causes her to have "wild mood swings and reckless behavior" according to her husband. She was diagnosed a year ago and since has been treated with several medications. Her symptoms today consist of nausea, vomiting, fatigue, tremor, and hyperreflexia. Lab results show an elevated BUN and creatinine, low sodium and elevated drug levels. All other results are normal. What medication is most likely the cause of her symptoms?

Lithium (Lithobid) Any sodium loss results in increased lithium levels. Signs and symptoms include vomiting and diarrhea which exacerbate the problem. Tremors, muscle weakness, confusion, vertigo, ataxia, hyperreflexia, rigidity, seizures, and coma may also be present.

A patient is evaluated for an overblown sense of self-importance and sense of entitlement. He is described as being arrogant, envious, exploitative and lacking in empathy. He is prone to mood swings and though he has a productive career in business he has few meaningful personal relationships. What is the most appropriate management for this patient?

Lithium (Lithobid) The diagnosis of narcissistic personality disorder is indicated in this scenario. His success in his career helps to lessen the probability of schizophrenia and/or delusions of grandeur. Antipsychotics are therefore not indicated and anxiolytics would not be helpful. Lithium can be used with this diagnosis if mood swings are prominent along with antidepressants to address the frequent co-morbid depression.

A patient with known drug dependence mentions that he commonly sees sounds and hears colors. What is his drug of choice?

Lysergic acid diethylamide Lysergic acid diethylamide is LSD, the phenomenon that this patient is experiencing is called synesthesia, which is common in patients who abuse hallucinogens.

A 9-year-old male is brought in by his mother who reports the patient has exhibited an extremely negative attitude for the past year. He seems angry much of the time and frequently loses his temper. Arguing over even trivial details is commonplace and he seems to take delight in annoying his family. His grades and conduct at school remain excellent. He has few friends, though he has never been seen bullying or destroying others' property. What is the most likely diagnosis?

Oppositional defiant disorder Oppositional defiant disorder (ODD) best fits this scenario and is differentiated from conduct disorder by the lack of bullying and the lack of destruction of property. Many children with ODD do drift into conduct disorders over time. His good grades and conduct at school lessen the probability of untreated ADD. Personality disorders (i.e. antisocial personality disorder) can not be diagnosed at this early an age.

On performing a mental status examination you notice that the patient tends to repeat words and phrases out of context to your questions. This type of thought disorder is defined as

Perseveration Definition of perseveration is the persistent repetition of words or concepts in the process of speaking. This is often seen in cognitive disorders, schizophrenia, and other mental illness.

A 28-year-old male presents with a recent onset of upsetting dreams, anxiety, and disturbing flashbacks. The patient reports that he returned from active duty in Iraq 3 months ago and was adjusting to life back at home until about 7 weeks ago when he began having an intense fear of loud noises and seeing sudden flashbacks to bombing attacks he endured during his time in Iraq. He had to take a leave from his army instructor duties as he was not able to handle the familiar settings of practice shooting ranges and sudden loud explosions during battalion training. After refusing to leave his house, he was finally persuaded by his wife to visit your clinic. What is the most likely diagnosis?

Posttraumatic stress disorder A patient complaining of flashbacks, nightmares, and avoidance of specific stimuli is likely suffering from posttraumatic stress disorder (PTSD). PTSD is often seen in war veterans, rape victims and survivors of traumatic events. PTSD symptoms do not have to manifest right after the event, but they must last for longer than 1 month to qualify as PTSD. Acute stress disorder has the same symptom profile as PTSD but lasts from 3 days to a month. The most common symptoms of PTSD are flashbacks, nightmares, avoidance of stimuli, anhedonia, insomnia, and suicidal thoughts. The main course of treatment is psychotherapy and the use of selective serotonin reuptake inhibitors. Once diagnosed, benzodiazepines and atypical antipsychotics should be avoided due to the overwhelming number of side effects. Caretakers should be wary of substance abuse and mood disorders.

A 24 year-old male presents to the emergency room via ambulance. He was found by the police walking naked on the highway. His speech is minimal but disorganized and he appears to be responding to auditory hallucinations. With further questioning, he is preoccupied with the delusion of the FBI listening in on his conversations. His mother tells you that he has been acting bizarre for two months now. What is the most likely diagnosis?

Schizophreniform disorder Schizophreniform disorder is characterized by the same features as schizophrenia except the total duration of the illness is at least one month and less than six months.

A 33 year-old hypertensive patient presents with increasing feelings of worthlessness and hopelessness over the past several months. Other symptoms include frequent crying episodes, loss of appetite, weight loss, insomnia with daytime sleepiness, and thoughts of suicide without any specific plans in place at this time. What would be the most appropriate first-line medication for this patient?

Selective serotonin reuptake inhibitors are the drugs of first choice for treating major depression.

A 17 year-old patient presents to the emergency department with agitation and hallucinations, and has one seizure. He admits to using "some drugs" but does not know what they were. On physical examination, temperature is 103 degrees F, BP 140/90, pulse 120, respirations 20. Remainder of the examination is unremarkable. Which diagnostic study will be of most help in managing this patient?

Serum creatinine kinase Serum creatinine kinase is the most sensitive test to detect rhabdomyolysis, a serious complication of seizures and hyperthermia related to drug abuse

What patient demographic is at highest risk for the development of anorexia nervosa?

Single, 20 year old, homosexual male Homosexual males are at increased risk for anorexia nervosa due to the strong community desire for slimness. This is not the case in the lesbian community. Wrestlers are at the greatest risk group among athletes along with ballet dancers. Though anorexia nervosa is more common in the female population, its onset is far more typical in the teens to early 20's.

A 25 year-old female graduate student presents to the student health center for the eighth time in three weeks to be sure she does not have meningitis. She read that there was a student on campus who had meningitis last month, and now she has headaches and is requesting to be tested to make sure she does not have meningitis. She has been evaluated at each visit, and physical examination has been completely normal each time. What is the most likely diagnosis?

Somatic symptom disorder Some previously distinct somatic disorders—somatization disorder, undifferentiated somatoform disorder, hypochondriasis, and somatoform pain disorder—are now considered somatic symptom disorders. All have common features. The symptoms may or may not be associated with another medical problem; symptoms no longer have to be medically unexplained. Sometimes the symptoms are normal body sensations or discomfort that do not signify a serious disorder. Patients are commonly unaware of their underlying mental problem and believe that they have physical ailments, so they typically continue to pressure physicians for additional or repeated tests and treatments even after results of a thorough evaluation have been negative.

A 17 year-old female is seen who has a history of eating large amounts of food at night 3 to 4 times weekly. These episodes are always followed by extreme guilt and either induced vomiting or hours of strenuous exercise. She excels at work and school and maintains many active relationships. Her physical examination reveals a normal BMI and an otherwise normal exam. What is the best initial intervention for this patient?

Start an antidepressant Uncomplicated bulimia generally does not require hospitalization as the physical manifestations encountered in bulimia are generally mild if present at all. Some patients with bulimia have concomitant substance abuse issues but her successful relationships and work activities lessen the probability. Antidepressants, not anxiolytics, have been found helpful in lessening the binge/purge cycles and improving overall well-being even outside of comorbid mood disorders.

A patient is started on a new antipsychotic medication for his disorder. Three days later he develops altered consciousness, lead-pipe rigidity, diaphoresis and catatonia. Vital signs reveal respiratory rate of 20, temperature of 105.6 degrees F, and pulse oximetry of 95% room air. What would be the most appropriate initial intervention in this patient?

Supportive care with fluids and antipyretics Neuroleptic malignant syndrome is characterized by extrapyramidal signs, blood pressure changes, altered consciousness, hyperpyrexia, muscle rigidity, dysarthria, cardiovascular instability, fever, pulmonary congestion and diaphoresis. Controlling fever and fluid support are the best initial management. With a normal pulse oximetry mechanical ventilation is not indicated.

A 25 year-old female presents with signs and symptoms of depression. She does not have any other known medical problems. What diagnostic study is indicated in the initial evaluation of this patient?

Thyroid stimulating hormone (TSH) Patients who are presenting with symptoms of depression should be evaluated with a TSH because 10 percent of patients evaluated for depression have previously undetected thyroid dysfunction.

UDS Opioids

Time: 1-3 days Detects: Codeine or morphine False +: poppy seeds (rare with raised threshold), Dextromethorphan, Quinolones, Rifampin

What is the most appropriate management of acute psychosis in a patient with schizophrenia?

There are numerous options used in the management of acute psychosis including benzodiazepines and antipsychotics such as haloperidol (first generation - typical -antipsychotic), risperidone or aripiprazole (second Generation - atypical - antipsychotics). Antidepressants and lithium have no benefit in the treatment of acute psychosis.

What class of antidepressants is associated with anticholinergic side effects, including cardiac dysrhythmias, dry mouth, sedation, and orthostatic hypotension?

Tricyclic antidepressants TCA's have well known anticholinergic effects.

What dietary substance interacts with monoamine oxidase-inhibitor antidepressant drugs?

Tyramine Monoamine oxidase inhibitors are associated with serious food/drug and drug/drug interactions. Patient must restrict intake of foods having a high tyramine content to avoid serious reactions. Tyramine is a precursor to norepinephrine. Some foods high in tyramine include: Aged cheeses, such as aged cheddar and Swiss; blue cheeses such as Stilton and Gorgonzola; and Camembert. Cheeses made from pasteurized milk are less likely to contain high levels of tyramine, including American cheese, cottage cheese, ricotta, farm cheese and cream cheese. Cured meats, which are meats treated with salt and nitrate or nitrite, such as dry-type summer sausages, pepperoni and salami. Fermented cabbage, such as sauerkraut and kimchee. Soy sauce, fish sauce and shrimp sauce. Yeast-extract spreads, such as Marmite. Improperly stored foods or spoiled foods. Broad bean pods, such as fava beans.

A 22 year-old female presents to the emergency department with rapid heart rate. She appears quite thin and dehydrated. She denies that she is thin, stating "I am so fat that I can hardly stand myself! That is why I exercise every day." She runs twelve to fifteen miles a day, and on weekends also bicycles forty to fifty miles. Her LMP was six months ago. On exam, she is 5' 6" tall and weighs 98 pounds. Temp 98 degrees F, pulse 100, respirations 18, BP 98/60. EKG shows sinus tachycardia. Laboratory findings include Na 138 mEq/L, K 2.8 mEq/L, Cl 91 mEq/L, BUN 35 mg/dL, Creatinine 1.1 mg/dL. What is the next most appropriate treatment?

Weight restoration and family therapy Anorexia nervosa requires a comprehensive, multidisciplinary approach to treatment that integrates medical management, individual psychotherapy, and family therapy. Currently, the best results have been shown with weight restoration accompanied by family therapy for patients with adolescent-onset anorexia nervosa and individual therapy for patients with onset after 18 years of age. Inpatient treatment is often required.

A 48 year-old alcoholic, whose last drink was 6 days ago, presents to the clinic complaining of palpitations and intermittent abdominal pain. On physical examination, blood pressure is 170/110 mm Hg, pulse 124/min, respirations 22/min, and temperature 100.4 degrees F. The patient is agitated and excitable. Cardiovascular examination reveals tachycardia without murmurs, gallops, or rubs, and is otherwise normal. What is the most appropriate intervention?

admit to the hospital for alcohol withdrawal This patient is exhibiting the signs and symptoms of alcoholic withdrawal characterized by delirium, autonomic hyperactivity, perceptual distortions, and fluctuating levels of psychomotor activity. Seizures are a common occurrence prior to DTs, but the delirium may occur without preceding seizures. This is a medical emergency, and if untreated, it has a mortality rate of 20%.

Rates of alcohol use in the adolescent are reportedly higher in:

adoptive children whose biological parent is an alcoholic There is a three-to fourfold increase in risk for alcohol dependence in adopted children whose biological parents are alcohol dependents.

A 54 year-old patient has acute onset of palpitations, tremulousness, profuse sweating, shortness of breath, and numbness and tingling of the extremities. Physical examination reveals a pulse of 104 beats/min and regular, respiratory rate of 30/min, blood pressure of 160/95 mm Hg. Arterial blood gases reveal a low pCO2. ECG shows no acute changes. The most likely diagnosis is

anxiety neurosis. Anxiety is characterized by short-lived, recurrent, unpredictable episodes of intense anxiety accompanied by marked physiological manifestations.

A 26 year-old female has a long history of sexual promiscuity and substance abuse. She frequently expresses anger when she feels abandoned. She also has difficulty in controlling her anger at times. Her past relationships have been intense and short-lived. She has attempted suicide twice in the past 18 months. This patient exhibits which disorder?

borderline personality Borderline personality disorder is characterized by instability of interpersonal relationships, marked impulsivity that is potentially self-damaging, inappropriate, intense anger or control of anger, recurrent suicidal attempts, gestures or threats, and identity disturbances. All of these are exhibited in this patient.

What effect result from cigarette smoking and contributes to atherogenesis?

chronic inflammation

Phenothiazines exert their antipsychotic effects by blocking

dopamine receptors. Blockage of the dopamine receptors in the mesolimbic areas of the brain is responsible for the antipsychotic effects of the phenothiazines; blockage of the dopamine receptors in the nigrostriatal areas lead to the motor side effects, such as dystonia and akathisia.

A patient presents within one hour of ingesting 30 tablets of diazepam (Valium). What is the most appropriate intervention?

gastric lavage Gastric lavage, along with the administration of activated charcoal and monitoring of vital signs and CNS status is the mainstay of therapy in the person who has overdosed on benzodiazepines. Vomiting should be induced in the person who is not comatose. Flumazenil (Romazicon), a specific benzodiazepine antagonist, might be used with caution in certain patients.

A 19 year-old female presents with complaints of intermittent abdominal pain associated with recent, frequent episodes of regurgitation of food for the past several months and worsening over the past 12 hours. She maintains a normal weight for her height however she seems obsessed with losing weight. On examination the physician assistant notes multiple dental caries, bilateral tenderness of the parotid glands and mild epigastric tenderness. What would you expect to find on laboratory tests to support your suspected diagnosis?

hypokalemia This patient most likely has bulimia nervosa - purging type. Self-induced vomiting is the most common method of purging and this is supported by the physical examination findings noted in this patient. Laboratory findings to support this diagnosis include hypochloremia with subsequent hypokalemia due to renal compensatory mechanisms, hypomagnesemia and metabolic alkalosis.

A 19 year-old woman has been consuming up to six beers daily since she was 16. She is now pregnant with her first child, has had little prenatal care, and is due to deliver in four weeks. Which neonatal problem should you anticipate?

low birth weight Fetal alcohol syndrome is a common cause of low birth weight.

A 53 year-old man with a history of hypertension is being treated with atenolol (Tenormin). He currently presents complaining of chronic fatigue, insomnia, decreased appetite, and difficulty concentrating for the past 3 weeks. His wife also notes that he no longer goes bowling with his friends and has lost interest in any sexual intimacy. Physical examination is unremarkable. What is the most likely diagnosis?

major depression Diagnostic criteria for a major depressive disorder include a loss of pleasure in usual activities, vegetative or physical changes (poor appetite, loss of energy), and cognitive changes such as difficulty in concentrating.

A 56 year-old man is admitted to a hospital unit for evaluation of rectal bleeding and weight loss. He has a strong family history of cancer. Soon after admission, a barium enema is scheduled. The patient refuses the "prep" because he fears x-ray radiation. He states he has had previous x-rays, but becomes frightened at the thought of an x-ray and "can't face it." The most likely diagnosis is

phobic neurosis. Phobic neurosis is a phobic ideation of displacement where the patient transfers feelings of anxiety from the object to one that can be avoided.

A 28 year-old male patient is being treated for depression and has been taking paroxetine (Paxil) for the past two and a half months with a marked improvement in symptoms. The patient reports problems with sexual functioning which he believes is related to the medication. What is an immediate concern with abrupt discontinuation of the medication?

risk of drug withdrawal symptoms The greatest risk of abrupt withdrawal of short acting SSRIs is a withdrawal syndrome or withdrawal delirium which represents cholinergic rebound.

A 65 year-old patient has a long history of schizophrenia that is treated with phenothiazines. On an unrelated clinic visit, the patient has difficulty sticking out her tongue, facial tics, increased blink frequency, and lip-smacking behavior. These involuntary movements are most suggestive of

tardive dyskinesia. Tardive dyskinesia is characterized by abnormal involuntary movements of the face, mouth, tongue, trunk, and limbs and may develop after months or years of treatment with neuroleptic drugs.


Kaugnay na mga set ng pag-aaral

Series 7 Quiz/Progress Exam Questions to Know

View Set

Perry Ch 7-10 Practice Questions

View Set

forensic science - DNA assessment

View Set

Algebra 1 Semester 2 Final Review - Part 1 (Systems & Exponents)

View Set

Helpdesk: Organize Your Computer: File Management (10/10)

View Set

Price Elasticity of Supply (PES)

View Set

Disorders Common Among Children and Adolescents - Ch 14

View Set